Angle Tangled

Geometry Level 5

There is a point P P inside a triangle A B C \triangle ABC such that P B A = P C A {\angle PBA=\angle PCA} .

Let A P C = α \angle APC=\alpha . Find the conditions on the sides a , b , c a,b,c of the triangle such that a minimum of α \alpha exists.

Let α min \alpha_\text{min} be the minimum value of α \alpha (if exists).

Submit your answer as the value of sin ( α min ) \sin(\alpha_\text{min})

c 2 b 2 \dfrac{c^2}{b^2} b 2 c 2 \dfrac{b^2}{c^2} c b \dfrac{c}{b} b c \dfrac{b}{c}

This section requires Javascript.
You are seeing this because something didn't load right. We suggest you, (a) try refreshing the page, (b) enabling javascript if it is disabled on your browser and, finally, (c) loading the non-javascript version of this page . We're sorry about the hassle.

4 solutions

Mark Hennings
Aug 3, 2020

As I have shown elsewhere , the locus of the points P P such that P B A = P C A \angle PBA = \angle PCA is a rectangular hyperbola H \mathcal{H} which is the isogonal conjugate of the perpendicular bisector L \mathcal{L} of the line B C BC . The section of the hyperbola which is internal to the triangle A B C ABC is the isogonal conjugate of the segment M N MN of L \mathcal{L} which is internal to A B C ABC . If P P is a point on H \mathcal{H} , let Q L Q \in \mathcal{L} be its isogonal conjugate. If we denote φ = A P C \varphi = \angle APC and θ = A Q C \theta = \angle AQC , then we note that 18 0 = φ + α + β 180^\circ = \varphi + \alpha + \beta and B + α + β + 36 0 θ = 36 0 B + \alpha + \beta + 360^\circ - \theta = 360^\circ , so that θ = 18 0 + B φ \theta = 180^\circ + B - \varphi .

Thus to minimize φ \varphi we need to maximize θ \theta . It is clear that the maximum value θ 0 \theta_0 of θ \theta will occur when we construct a circle that passes through both A A and C C , and which is tangential to the line L \mathcal{L} . The centre X X of this circle can be found as the point of intersection of the perpendicular bisector of A C AC with the parabola P \mathcal{P} with C C as focus and L \mathcal{L} as directrix. Since B P C = 18 0 ( B α ) ( C α ) = A + 2 α \angle BPC = 180^\circ - (B - \alpha) - (C - \alpha) = A + 2\alpha , this means that, in the case that θ \theta is minimized, B P A = 9 0 \angle BPA = 90^\circ , and hence sin φ 0 b = sin α A P = 1 c \frac{\sin \varphi_0}{b} = \frac{\sin\alpha}{AP} \; = \; \frac{1}{c} so that sin φ 0 = b c \sin\varphi_0 = \boxed{\tfrac{b}{c}} .

Thus, if φ \varphi is minimized at some point internal to the triangle, then the minimum value of sin φ \sin\varphi is b c \tfrac{b}{c} . There are three conditions that the triangle has to satisfy for this minimum to be achieved in the desired manner, and they can most easily be expressed in terms of the angles of the triangle:

  • B < C B < C ,
  • C C is acute,
  • C A M \angle CAM is acute ( M M is the midpoint of B C BC ).

If B > C B > C then A A is on the same side of L \mathcal{L} as B B , and hence the maximum value of θ \theta clearly occurs when Q = N Q = N , which now lies on the edge A C AC . For A P B \angle APB to be equal to 9 0 90^\circ (which is necessary if the maximum value of θ \theta occurs inside A B C ABC ), we must have C C an acute angle, because A C B < A P B \angle ACB < \angle APB . Assume now that B < C < 9 0 B < C < 90^\circ . If C A M \angle CAM is obtuse, then A A lies inside the circle with diameter 1 2 a \tfrac12a passing through M M and C C . Let A A' be the intersection of the extension of A C AC with this circle. Then the perpendicular bisector of A C A'C passes through the centre of this circle, which is the vertex of the parabola P \mathcal{P} . The perpendicular bisector of A C AC is parallel to this other perpendicular bisector, but crosses the the B C BC "below" the vertex of the parabola. Thus the perpendicular bisector of A C AC meets the parabola at a point X X which is to the "left" of B C BC , and hence Q Q lies outside the line segment M N MN . Conversely, if C A M \angle CAM is acute, then A A lies outside this circle, and Q Q lies inside M N MN .

Using the Cosine Rule, and the known formula for the length of a triangle median, the conditions that B < C B < C , C < 9 0 C < 90^\circ and C A M < 9 0 \angle CAM < 90^\circ can be written in terms of the sides of the triangle as b < c b < c , c 2 < a 2 + b 2 c^2 < a^2 + b^2 and a 2 < 3 b 2 + c 2 a^2 < 3b^2 + c^2 , respectively.

Atomsky Jahid
Jul 29, 2020

Let P B A = P C A = θ , B P C = x = A + 2 θ , A P B = β = 2 π x α \angle PBA = \angle PCA = \theta , \> \angle BPC = x = \angle A + 2 \theta , \> \angle APB = \beta = 2 \pi - x - \alpha .

Applying the sine rule for Δ A P B , Δ A P C \Delta APB, \> \Delta APC , we get sin β c = sin θ A P = sin α b c sin α = b sin ( 2 π x α ) c sin α = b sin ( x + α ) c sin α = b sin x cos α b cos x sin α tan α = b sin x c + b cos x α = tan 1 ( b sin x c + b cos x ) d α d x = b 2 b c cos x ( c + b cos x ) 2 + ( b sin x ) 2 \begin{aligned} \frac{\sin \beta}{c} &=\frac{\sin \theta}{AP} = \frac{\sin \alpha}{b} \\ c \sin \alpha &= b \sin (2 \pi -x -\alpha) \\ c \sin \alpha &= - b \sin (x +\alpha) \\ c \sin \alpha &= - b \sin x \cos \alpha -b \cos x \sin \alpha \\ \tan \alpha &= \frac{-b \sin x}{c+b \cos x} \\ \implies \alpha &= \tan^{-1} \Big(\frac{-b \sin x}{c+b \cos x} \Big) \\ \therefore \frac{ d\alpha}{dx} &= \frac{-b^2-bc \cos x}{(c + b \cos x)^2+(b \sin x)^2} \end{aligned}

For an extremum, d α d x = 0 b 2 b c cos x ( c + b cos x ) 2 + ( b sin x ) 2 = 0 b 2 b c cos x = 0 cos x = b c \begin{aligned} \frac{ d\alpha}{dx} &= 0 \\ \frac{-b^2-bc \cos x}{(c + b \cos x)^2+(b \sin x)^2} &= 0 \\ -b^2-bc \cos x &= 0 \\ \implies \cos x &= - \frac{b}{c} \end{aligned}

So, tan α = b sin x c + b cos x = b ( c 2 b 2 c ) c + b ( b c ) = b c 2 b 2 sin α = b c \begin{aligned} \tan \alpha &= \frac{-b \sin x}{c+b \cos x} \\ &= \frac{-b (\frac{ \sqrt{c^2-b^2} }{c}) }{c+b (- \frac{b}{c})} \\ &= - \frac{b}{ \sqrt{c^2-b^2} } \\ \therefore \sin \alpha &= \frac{b}{c} \end{aligned}

As we can see from the expression for tan α \tan \alpha , there is a radical c 2 b 2 \sqrt{c^2-b^2} . So, for an extremum to exist, we need c > b c > b . Also, we need the point P P to stay inside Δ A B C \Delta ABC . For that we need to satisfy 0 < θ < B 0 < \theta < \angle B . We can also write this in terms of x x to get A < x < A + 2 B \angle A < x < \angle A + 2 \angle B . This inequality can be written in terms of side lengths as c 2 b 2 < a 2 < c 2 + 3 b 2 c^2 -b^2 < a^2 < c^2 + 3b^2 .

You might like to look at my solution to see where the conditions on a , b , c a,b,c come from.

Mark Hennings - 10 months, 1 week ago

Log in to reply

I was able to get a 2 < c 2 + 3 b 2 a^2 < c^2 + 3b^2 from 0 < θ 0 < \theta . For c 2 b 2 < a 2 c^2 - b^2 < a^2 , I had to prove C < π 2 \angle C < \frac{\pi}{2} using θ < B C < B + cos 1 b c \theta < \angle B \implies \angle C < \angle B + \cos^{-1} \frac{b}{c} . I drew a circle of length c c around A A to prove it.

I am going through your solution, although I am not familiar with many of the terms that you used.

Atomsky Jahid - 10 months, 1 week ago
Chew-Seong Cheong
Jul 27, 2020

Let P B A = P C A = θ \angle PBA = \angle PCA = \theta . Then B P C = 18 0 ( B θ ) ( C θ ) = 18 0 B C = 2 θ = A + 2 θ \angle BPC = 180^\circ - (B-\theta) - (C-\theta) = 180^\circ - B - C = 2\theta = A + 2\theta and A P B = 36 0 A 2 θ α \angle APB = 360^\circ - A - 2\theta - \alpha . Let A P = d AP = d . By sine rule ,

sin α b = sin θ d = sin ( 36 0 A 2 θ α ) c sin α b = sin ( A + 2 θ + α ) c c sin α = b sin ( A + 2 θ + α ) \begin{aligned} \frac {\sin \alpha}b & = \frac {\sin \theta}d = \frac {\sin (360^\circ - A - 2\theta -\alpha)}c \\ \implies \frac {\sin \alpha}b & = \frac {-\sin (A + 2\theta + \alpha)}c \\ c \sin \alpha & = - b \sin (A+2\theta + \alpha) \end{aligned}

To find the minimum α \alpha , we first find the θ \theta such that d α d θ = 0 \dfrac {d\alpha}{d\theta} = 0 .

c cos α d α d θ = b cos ( A + 2 θ + α ) ( 2 + d α d θ ) d α d θ = 2 b cos ( A + 2 θ + α ) c cos α + b cos ( A + 2 θ + α ) \begin{aligned} c \cos \alpha \frac {d\alpha}{d\theta} & = - b \cos (A + 2 \theta + \alpha) \left(2 + \frac {d\alpha}{d\theta} \right) \\ \implies \frac {d\alpha}{d \theta} & = \frac {-2b\cos (A+2\theta+\alpha)}{c \cos \alpha + b \cos(A+2\theta + \alpha)} \end{aligned}

We note that d α d θ = 0 \dfrac {d\alpha}{d\theta} = 0 , when A + 2 θ + α = 27 0 A+2\theta + \alpha = 270^\circ , since the angle is larger than 9 0 90^\circ . Since

c sin α = b sin ( A + 2 θ + α ) sin α min = b sin 27 0 c = b c \begin{aligned} c \sin \alpha & = - b \sin (A+2\theta + \alpha) \\ \implies \sin \alpha_{\min} & = \frac {-b \sin 270^\circ}c = \boxed{\frac bc} \end{aligned}

The minimum doesn't always exist. Can you figure out the conditions on the sides such that the minimum exists?

Digvijay Singh - 10 months, 2 weeks ago

Log in to reply

In any configuration of a , b , c a, b, c , there will be a range of values for α \alpha . And as long as there is a range of values, there will be a maximum and a minimum.

Atomsky Jahid - 10 months, 2 weeks ago

Log in to reply

Not necessarily. For example, what is the minimum value in the range ( 0 , 1 ) (0,1) ? It doesn't exist.

Digvijay Singh - 10 months, 2 weeks ago

Log in to reply

@Digvijay Singh Well, you can then ask for infimum or supremum.

Atomsky Jahid - 10 months, 2 weeks ago

Log in to reply

@Atomsky Jahid Yes, but I asked for a minimum here, which surely exists only when c > b c>b and c 2 b 2 < a < c 2 + 3 b 2 \sqrt{c^2-b^2}<a<\sqrt{c^2+3b^2}

Digvijay Singh - 10 months, 2 weeks ago

Log in to reply

@Digvijay Singh Am I getting the definitions wrong? The maximum attainable value for sin α \sin \alpha is b c \frac{b}{c} . Are you implying sin α \sin \alpha has a constant value when those conditions are met?

Atomsky Jahid - 10 months, 2 weeks ago

Log in to reply

@Atomsky Jahid yes, when these conditions are met, then there is unique point P P such that α \alpha attains a minimum value which is π sin 1 ( b c ) \pi-\sin^{-1}\left(\frac{b}{c}\right) .

Digvijay Singh - 10 months, 2 weeks ago

Log in to reply

@Digvijay Singh Thanks. It looks like I confused α \alpha with sin α \sin \alpha . I will have another look at the problem.

Atomsky Jahid - 10 months, 2 weeks ago

The conditions for a minimum to exist are:

  • c > b c>b

  • c 2 b 2 < a < c 2 + 3 b 2 \sqrt{c^2-b^2}<a<\sqrt{c^2+3b^2}

Digvijay Singh - 10 months, 2 weeks ago
David Hairston
Jul 27, 2020

Note that as Point P moves toward Point B, angle alpha decreases. Also, side b is opposite angle alpha. For the previous considerations, side c eventually approximates the hypotenuse of the minimum angle alpha. Q.E.D.:, by definition, the sine of angle alpha is opposite over hypotenuse, or b / c.

BTW, I'm not sure I could have deduced an answer if b / a was a distractor ...

0 pending reports

×

Problem Loading...

Note Loading...

Set Loading...